¿Dónde está el resto? - Ecuación de Schrödinger para una partícula libre cuya posición se conoce

suposiciones

  • Estaré trabajando en una dimensión espacial en todo momento. Debería ser posible generalizar a tres dimensiones con relativa facilidad, pero hacerlo agrega poco a la conversación y solo sirve para dificultar el cálculo.

  • Las partículas elementales no tienen necesariamente medida ni estructura interna alguna. Esto significa que si una partícula elemental alguna vez tiene una posición, entonces esa partícula existe solo en un solo punto en el espacio. De esto, podemos inferir que si la posición de una partícula en el tiempo t se sabe que es y , entonces su función de onda ψ debe ser tal que | ψ ( X , t ) | 2 = d ( X y ) .

  • La velocidad de una partícula con masa en reposo positiva, medida en cualquier marco de referencia, siempre es menor que C , la velocidad de la luz. Por lo tanto, si la posición de una partícula [masiva] es y en el momento t y y en el momento t , entonces | y y t t | < C .

Notación

  • porque la carta ϕ se usa comúnmente tanto para la fase como para el impulso, me gustaría usar φ para representar la fase y ϕ para representar el impulso.

El problema

Supongamos que medimos la posición de una partícula libre, asignando a esta posición un valor de 0 y así mismo marcando el tiempo como 0 . Entonces tenemos la función de densidad de probabilidad para la posición de la partícula dada por...

| ψ ( X , 0 ) | 2 = d ( X )

...de lo que podemos inferir...

(ecuación 1) ψ ( X , 0 ) = d ( X ) mi i φ 0

dónde φ 0 es la fase inicial de ψ .

A partir de aquí, deseamos predecir la posición de la partícula en un tiempo futuro arbitrario t . Para hacerlo, debemos resolver la ecuación de Schrödinger dependiente del tiempo...

(ecuación 2) i t ψ ( X , t ) = 2 2 metro 2 X 2 ψ ( X , t )

...con condición inicial (eq.1). Afortunadamente, ya existe una solución a este problema.

Primera solución

La primera solución la da Emilio Pisanty aquí :

ψ ( X , t ) = { d ( X ) t = 0 metro 2 π t Exp [ i ( metro X 2 2 t π 4 ) ] t > 0

Algo problemático, esta solución no es normalizable (veremos por qué esto es problemático más adelante). Sin embargo, indica que 1) debido a la incertidumbre, la posición de la partícula en el tiempo t > 0 siempre es desconocido y 2) a medida que pasa el tiempo, la probabilidad de encontrar la partícula cerca de su ubicación original disminuye, como si se "esparciera" hasta el infinito. Este segundo punto proporciona algo de la intuición detrás del razonamiento de esta pregunta.

Segunda "Solución"

La segunda solución, o mejor dicho, la mitad de la solución, es el resultado de mi débil intento de encontrar una solución normalizable. Para empezar, podemos considerar nuestra condición inicial como el caso límite de una distribución gaussiana...

| ψ ( X , t ) | 2 = 1 σ ( t ) 2 π Exp ( X 2 2 σ ( t ) 2 )

...dónde σ ( 0 ) = 0 . De esto tenemos...

ψ ( X , t ) = 1 σ ( t ) 2 π Exp ( X 2 4 σ ( t ) 2 + i φ ( t ) )

Podemos retroalimentar esto en (ecuación 2), omitiendo la transformación habitual hacia y desde la base de impulso, para obtener...

(ecuación 3) X σ ( t ) 2 i metro X 2 t σ ( t ) + 2 i metro σ ( t ) 2 t σ ( t ) + 4 metro σ ( t ) 3 t φ ( t ) = 0

...que luego podemos resolver para pares de funciones ( φ , σ ) que cumplen las condiciones deseadas. Es necesario resolver en la base de la posición porque la transformación de una solución a partir de la base del momento producirá nuevamente una solución no normalizable. Siempre que las soluciones a {(eq.3), σ ( 0 ) = 0 } existen (estoy bastante seguro de que no, pero no estoy escribiendo la prueba), podemos usar esto para engañar a la incertidumbre evitando el impulso por completo.

Por qué ambas soluciones (y casi todas las demás soluciones) son incorrectas

En ambos casos, sabemos que la ubicación exacta de la partícula en el tiempo t = 0 es 0 . Esto significa que para que la partícula sea observada en alguna posición X en el momento t , debe poseer, en algún momento, una velocidad de al menos | X / t | .

Debido a que esta velocidad no puede exceder C , incluso si no tenemos idea de dónde está la partícula , sabemos que no está fuera de la región ( C t , C t ) . Por lo tanto, en cualquier momento t , ψ ( X , t ) = 0 para todos excepto para muchos contables X ( C t , C t ) .

Este no es el caso si ψ no es normalizable (como en la solución 1), y no necesariamente el caso incluso cuando ψ es (como en la solución 2, ostensiblemente), pero requiere estrictamente que el apoyo esencial de ψ ser compacto Específicamente, tenemos la condición de que apoyar ( ψ ( , t ) ) [ C t , C t ] . Esta condición no es impuesta de manera obvia por (eq.2), lo que me lleva a lo siguiente:

Cualquiera

  1. (eq.2) está incompleto. Se requieren términos y/o ecuaciones adicionales que imponen la restricción apoyar ( ψ ( , t ) ) [ C t , C t ] en soluciones siempre que se conozca con exactitud la posición inicial de la partícula. Además, tenemos que ψ ( X , t ) = 0 para todos X ( C t , C t ) cuando metro > 0 , para todos X [ C t , C t ] cuando metro = 0 , y posiblemente alguna otra condición cuando metro < 0 .

  2. Al menos una de mis suposiciones es incorrecta.

    2a. Debido a alguna peculiaridad de las matemáticas involucradas (por ejemplo, un producto cruzado oculto o cualquier cosa que involucre cuaterniones), las soluciones físicamente relevantes para la ecuación de Schrödinger solo son posibles en 3+1 dimensiones.

    2b. La noción de "posición", en el sentido clásico, no se aplica en absoluto a las partículas cuánticas. Incluso la medición de la posición de una partícula solo produce una función de densidad de probabilidad con un máximo ubicado en el punto en que se observó. En otras palabras, la condición inicial ψ ( X , 0 ) = d ( X ) mi i φ 0 es inválido.

    Esto implica que es posible observar directamente una partícula "individual" que aparece simultáneamente en dos puntos separados en el espacio, y que las partículas pueden detectarse a cualquier distancia (aunque la probabilidad de detección disminuye en función de esa distancia).

    2c. La mecánica cuántica permite violaciones al límite de velocidad cósmica. En este caso, tales violaciones deberían ser omnipresentes en distancias cortas y escalas de tiempo cósmicas. Estaría muy interesado en cualquier evidencia experimental de este fenómeno.

  3. Los dos anteriores.

o

  1. He descuidado algo muy fundamental para la mecánica cuántica, sin el cual esta pregunta está mal formulada y es irrelevante. Una vez que esté seguro de qué se trata (y no, no es la transformada de Fourier), puedo eliminar esta pregunta.

(Por supuesto, existe la posibilidad de literalmente cualquier otra cosa , pero esto sugiere la aterradora verdad de que en realidad no sabemos cómo funciona el universo; y dado que no tengo ganas de cuestionar la totalidad de la física moderna en este momento , supongamos que la mecánica cuántica es "principalmente correcta", o al menos un buen punto de partida).

¿Por qué cree que dos mediciones de posición sucesivas de "una partícula" deben estar en los conos de luz de cada uno (uno hacia adelante, uno hacia atrás)? Hay muchos diagramas de Feynman de una partícula dentro, una partícula fuera, donde la entrada y la salida de la "partícula única" están separadas como en el espacio. Además, varios resultados en túneles, por ejemplo, quantamagazine.org/… .

Respuestas (3)

Tenga cuidado con el uso de la palabra "medida" para pasar de contrabando un proceso físico mal definido o imposible. La noción de que podría haber una medición de posición que resulte en una función de onda de función delta es un ejemplo de una idea no física. Incluso si reemplazamos la función delta por una función de altura finita y ancho muy pequeño, todo el cálculo sigue siendo incorrecto porque ahora tiene suficiente energía cinética para formar pares de electrones y positrones. Entonces, el cálculo debe hacerse a través de la teoría cuántica de campos.

Se puede generalizar de la siguiente manera. Si suponemos que una partícula está confinada a una región de ancho a entonces la desviación estándar de su cantidad de movimiento tendrá un límite inferior por

Δ pag a
(Principio de incertidumbre de Heisenberg). Si la cantidad de movimiento media es cero, se deduce que la cantidad de movimiento al cuadrado es del orden ( / a ) 2 y por lo tanto la energía cinética es del orden
( γ 1 ) metro C 2
dónde
γ ( 1 ( a metro C ) 2 ) 1 / 2
dónde metro es la masa en reposo de la partícula. En el nivel de aproximación involucrado, no necesitamos preocuparnos por los detalles; el principio de incertidumbre de Heisenberg da el orden correcto de magnitud. Esta energía cinética es suficiente para proporcionar la energía para crear un par partícula-antipartícula si
( γ 1 ) metro C 2 2 metro C 2
es decir γ 3 . Esto sucede cuando a metro C , por lo que el resultado es que el confinamiento de partículas a una región más pequeña que
a metro C
siempre es cuestionable. Y 'confinamiento' aquí no necesita significar 'confinamiento permanente'; podría incluir cualquier proceso que causara temporalmente que la función de onda de la posición de la partícula se extendiera por / metro C o menos. La combinación h / metro C también se denomina "longitud de onda Compton" porque aparece en el efecto Compton, pero su papel aquí es quizás su primera importancia. (Ver wiki para más información).

Entonces, para repetir, una vez que estamos en este régimen, la teoría cuántica no relativista ya no es suficiente.

De manera similar, para explorar la noción de cambios más rápidos que la luz en las funciones de onda, para asegurarse de no cometer errores, debe acudir a una teoría relativista, y esa es una vez más la teoría cuántica de campos. Y la teoría cuántica de campos es diferente de la mecánica cuántica no relativista en formas profundas.

Sus respuestas 1 y 2b son aproximadamente correctas. La ecuación de Schrödinger que has escrito no es relativista. Para capturar la relatividad, debe usar la teoría cuántica de campos (esta es una especie de respuesta 4.

2b también es relevante porque generalmente no se piensa que las funciones delta de Dirac sean funciones de onda físicas de la misma manera que las ondas sinusoidales puras (infinitas en el tiempo, transformada de Fourier delta de Dirac) no son señales físicas. Es útil incluirlas en el formalismo matemático, pero solo las sumas infinitas de funciones delta de Dirac que se integran con alguna medida (por ejemplo, una función normal como una gaussiana) son funciones de onda físicas. Estos no tienen los mismos problemas de relatividad que planteas, pero aún pueden tener algunos.

Pero de nuevo, la pregunta solo se resuelve por completo cuando te refieres a qft. Allí, la función de modo y algunos conmutadores se definen de tal manera que se incorpora la adherencia a la relatividad especial.

Tal vez esta sea una pregunta aparte, pero si no consideramos el delta de Dirac como una función de onda viable, entonces, ¿qué sucede con la función de onda cuando medimos la posición de una partícula?
Si uno insiste en una interpretación basada en el colapso (no es mi preferencia, pero relevante para esta pregunta, lo que sucede es que nuestra medición siempre es 100% precisa. Debido a las limitaciones técnicas en cualquier experimento que realicemos, siempre habrá cierta incertidumbre en nuestra medición) de posición. El estado entonces no colapsará a un gaussiano, sino a algún estado que refleje nuestro nuevo conocimiento sobre el estado. La función de onda alcanza su punto máximo en alguna posición pero cae a una tasa finita dada por nuestra precisión de medición.
Puede buscar medidas con valores de operadores positivos (povm) para obtener más información sobre cómo los detalles de la medición informan el estado cuántico posterior a la medición.

La ecuación de Schrödinger (con el hamiltoniano utilizado aquí) es una ecuación no relativista, por lo que no obedece al límite de la velocidad de la luz.

Comenzando con una partícula en un estado propio de posición, contiene todos los valores de momento posibles: la transformada de Fourier de la función delta es una constante, es decir, es "igualmente probable" que tome cualquier número real como momento, en algún sentido (aunque esto es una función de onda espacial de impulso no normalizable, por lo que es bastante poco física). Entonces, hay cierta amplitud para que viaje arbitrariamente lejos en un período de tiempo arbitrariamente corto. En consecuencia, la función de onda del espacio de posición tiene soporte infinito para cualquier t > 0 .

Así que las soluciones no son realmente malas; son correctos dentro de los límites del marco matemático que está utilizando. Para hablar de partículas relativistas tendrías que intentar cambiar a un hamiltoniano relativista, y luego esto te lleva rápidamente al ámbito de la teoría cuántica de campos.

En la QFT relativista, se puede demostrar que los operadores de campo separados en forma de espacio conmutan, que es la forma en que QFT expresa la idea de que las influencias no pueden propagarse más rápido que la luz a través del campo (la llamada "microcausalidad").